0 Daumen
784 Aufrufe

Zu beweisen ist folgende Ungleichung:

\( \sqrt{\frac{x^{2}+y^{2}}{2}} \geq \frac{x+y}{2} \geq \sqrt{x \cdot y} \geq \frac{2}{\frac{1}{x}+\frac{1}{y}} \)

x,y in ℝ^{+}


Ansatz/Problem:

Habe probeweise für x und y jeweils 1 eingesetzt. Ist wahr.

Danach weiß ich leider nicht weiter.

Avatar von

Warum Induktion? \(x\) und \(y\) sind doch beliebige reelle Zahlen. Du kannst jede dieser Gleichungen durch umformen zeigen.

3 Antworten

+1 Daumen
 
Beste Antwort


Ich würde mir immer zwei Gleichungen (erst die ersten Beiden, dann die nächsten u.s.w.) und dann entsprechend umformen sodass auf beiden Seiten das Gleiche steht. Ein Beispiel, das ich an der Uni hatte:


Es seien a, b ∈ R, a+b ≠ 0 und ab ≥ 0. Zeigen Sie:


$$ {\frac{2ab}{a+b}}^{2} ≤ ab ≤ {\frac{a+b}{2}}^{2} $$


Nun hab ich mir eine Ungleichung davon genommen und umgeformt:



$$ ab ≤ {\frac{a+b}{2}}^{2} $$

$$ a+b  ≤ \frac{a^2 +2ab +b^2}{4} $$

$$ 4ab ≤ a^2 +2ab +b^2 $$

$$ 0 ≤ a^2 -2ab +b^2 $$

$$ 0 ≤ (a-b) ^2 $$


Dann hab ich das noch mit der ersten Ungleichung gemacht und dort kam dasselbe am Ende raus.


Dann hab ich noch eine Begründung geschrieben wann es in eine Gleichung über geht etc.


So musst du das hier auch machen :)


Avatar von
+2 Daumen

Hi,
man kann das auch verallgemeinern für \( n \ge 2 \) s.d. gilt
$$ \sqrt{ \frac{\sum_{i=1}^n x_i^2}{n } } \ge \frac{\sum_{i=1}^n x_i}{n} \ge \sqrt[n]{ \prod_{i=1}^n x_i } \ge \frac{n}{\sum_{i=1}^n \frac{1}{x_i} } $$
D.h das quadratische Mittel ist \( \ge \) als das arithmetisches Mittel ist \( \ge \) als das geometrische Mittel ist \( \ge \) als das harmonische Mittel.


Die erste Ungleichung beweist man so
$$  0 \le \sum_{i=1}^n ( x_i - x_{n+1} )^2 \le $$
$$ \sum_{i=1}^n x_i^2-2x_{n+1}\sum_{i=1}^nx_i+nx_{n+1}^2 + \underbrace{ n\sum_{i=1}^nx_i^2 - \left(  \sum_{i=1}^nx_i \right)^2 }_{I.V. \ge 0} +x_{n+1}^2 - x_{n+1}^2 $$
Daraus folgt
$$ 0 \le (n+1) \sum_{i=1}^{n+1} - \left( \sum_{i=1}^{n+1} a_i \right)^2 $$ q.e.d.


Die zweite Ungleichung folgt aus der Bernoullischen Ungleichung
$$  \left( \frac{ \overline{x_{n+1}}}{\overline{x_n}} \right)^{n+1} = \left( 1 + \frac{x_{n+1} - \overline{x_n}}{(n+1) \overline{x_n}}  \right)^{n+1} $$ und wegen der Bernoullischen Ungleichung folgt
$$ \left( \frac{ \overline{x_{n+1}}}{\overline{x_n}} \right)^{n+1} \ge \frac{x_{n+1}}{\overline{x_n}}  $$ und daraus folgt
$$ \overline{x_{n+1}}^{n+1} \ge \overline{x_n}^n x_{n+1} \underbrace{ \prod_{i=1}^{n} x_i x_{n+1}   }_{I.V.}= \prod_{i=1}^{n+1} x_i $$ q.e.d.

Die dritte Ungleichung folgt aus der zweiten, in dem man anstelle \( x_i \) den Ausdruck \( \frac{1}{x_i} \) einsetzt.

Avatar von 39 k
0 Daumen
(x+y) / 2 ≥ wurzel(x*y)

x+y  ≥  2* wurzel(x*y)   weil alles positiv ist, ist quadrieren eine Äquivalenzumformung !
(x+y)^2 ≥  2* xy
x^2 + 2xy + y^2 ≥  2* xy
x^2 + y^2 ≥  0  gilt sicher für alle x,y aus IR+.

ähnlich lassen sich auch die anderen Teile umformen.
Avatar von 288 k 🚀

oder bei dem ersten Teil:

erst mal quadrieren /Äquivalenz s. mein Lösungsvorschlag oben)

(x^2 + y^2 ) / 2   ≥ (x+y )^2 / 4

2x^2 + 2y^2   ≥ (x+y )^2

2x^2 + 2y^2   ≥ x^2+y  + 2xy + y^2

x^2 + y^2 ≥ 2xy

(x-y)^2 ≥ 0 also wahr.

Ein anderes Problem?

Stell deine Frage

Willkommen bei der Mathelounge! Stell deine Frage einfach und kostenlos

x
Made by a lovely community